LSAT and Law School Admissions Forum

Get expert LSAT preparation and law school admissions advice from PowerScore Test Preparation.

 Administrator
PowerScore Staff
  • PowerScore Staff
  • Posts: 8917
  • Joined: Feb 02, 2011
|
#41261
Complete Question Explanation
(The complete setup for this game can be found here: lsat/viewtopic.php?t=6641)

The correct answer choice is (C)

If the fifth solo is a TW block, then Template #2 is again in effect:
PT66_J12_Game_#4_#23_diagram 1.png
From the fourth rule, however, when W performs the fifth solo, then Z must perform the second solo:
PT66_J12_Game_#4_#23_diagram 2.png
The only uncertainty remaining in this solution is which pianist performs the third solo, and because this is a Could Be True question, you should immediately look for an answer that references the pianist for the third solo. Answer choice (C) offers Z as the pianist for the third solo, and since this could be true, answer choice (C) is therefore correct.
You do not have the required permissions to view the files attached to this post.
 karlaurrea
  • Posts: 20
  • Joined: Aug 26, 2012
|
#6606
Hello… Hopefully someone can help me understand the setup to be able to see things quicker and arrive to the correct answers and the majority of the questions for the last game on the June 2012 LSAT game questions 19-23; The more I try to get the correct answers I just don’t understand how they arrive to those answers…. (Hopefully my questions and explanations do not confuse anyone)


Option 1:
W w/z w/z Z
M T T M __
1 2 3 4 5

Option 2:
W z/w z/w W
M _ T T __
1 2 3 4 5



Question 23- I found this to be a very trick type question when comparing it to other question types and games. The question says “could be true” not “must be true” and as we all in the LSAT world that terminology is key when answering a question, because you can’t answer a “could be” with what “must be” and visa versa, yet if we diagram this question, YES I can totally see why Z would be second given that W is 5th and based on the rules Z would have to be second because the pianist on 2 can’t be in 5, but again, this question wasn’t about what “must be true” if not what could be true, and that case then answer choice D and E are definite could be true answers. So when do I know the appropriate way of answering these types of questions.

***I hope I have made sense, please get back to when possible, and most importantly out of this game, is I would really like to know how you would set it up on the first time to be able to see all inferences and move through the game quickly.

THANKS!!!
User avatar
 Dave Killoran
PowerScore Staff
  • PowerScore Staff
  • Posts: 5852
  • Joined: Mar 25, 2011
|
#6612
Hi Karla,

You are correct that the wording here is critical. Could Be True questions seek an answer that can occur, but does not necessarily have to occur. Thus, when you make a diagram in a Could question, if you see something uncertain, that is probably what will be tested!

Let's look at the diagram for this question (I won't walk through it, because I think you understand the diagram):


..... ..... ..... W ..... Z ..... ? ..... Z ..... W
..... ..... ..... M ..... T ..... T ..... M ..... T
..... ..... ..... 1 ..... 2 ..... 3 ..... 4 ..... 5

See that "?" on the third piece? That's the only thing that is unknown in this question. Either W or Z could perform the third piece, and thus, in a Could question like this one, I'd immediately look for "W performs third" or "Z performs third as the answer." The latter appears as (C), and thus answer choice (C) is correct.

As an aside, your explanation makes it appear as if you misread (C) to read: "Z performs the second solo." Misreading is always a killer (and that would probably explain why this looked like a trick question to you).

Please let me know if that helps. Thanks!
 karlaurrea
  • Posts: 20
  • Joined: Aug 26, 2012
|
#6624
Yes!! It is all clear now! Thanks so much :)

Get the most out of your LSAT Prep Plus subscription.

Analyze and track your performance with our Testing and Analytics Package.